Difference between revisions of "2004 AMC 8 Problems/Problem 6"

(Created page with "== Problem == After Sally takes <math>20</math> shots, she has made <math>55\%</math> of her shots. After she takes <math>5</math> more shots, she raises her percentage to <math>...")
 
Line 2: Line 2:
 
After Sally takes <math>20</math> shots, she has made <math>55\%</math> of her shots. After she takes <math>5</math> more shots, she raises her percentage to <math>56\%</math>. How many of the last <math>5</math> shots did she make?
 
After Sally takes <math>20</math> shots, she has made <math>55\%</math> of her shots. After she takes <math>5</math> more shots, she raises her percentage to <math>56\%</math>. How many of the last <math>5</math> shots did she make?
  
<math> \textbf{(A)}1\qquad\textbf{(B)}2\qquad\textbf{(C)}3\qquad\textbf{(D)}4\qquad\textbf{(E)}5 </math>
+
<math> \textbf{(A)}\ 1\qquad\textbf{(B)}\ 2\qquad\textbf{(C)}\ 3\qquad\textbf{(D)}\ 4\qquad\textbf{(E)}\ 5 </math>
  
 
== Solution ==
 
== Solution ==
Sally made <math>0.55*20=11</math> shots originally. Letting <math>x</math> be the number of shots she made, we have <math>\frac{11+x}{25}=0.56</math>. Solving for <math>x</math> gives us <math>x=3</math> <math>\boxed{\textbf{(C)}\ 3}</math>
+
Sally made <math>0.55*20=11</math> shots originally. Letting <math>x</math> be the number of shots she made, we have <math>\frac{11+x}{25}=0.56</math>. Solving for <math>x</math> gives us <math>x=\boxed{\textbf{(C)}\ 3}</math>
 +
 
 +
==See Also==
 +
{{AMC8 box|year=2004|num-b=5|num-a=7}}

Revision as of 04:33, 24 December 2012

Problem

After Sally takes $20$ shots, she has made $55\%$ of her shots. After she takes $5$ more shots, she raises her percentage to $56\%$. How many of the last $5$ shots did she make?

$\textbf{(A)}\ 1\qquad\textbf{(B)}\ 2\qquad\textbf{(C)}\ 3\qquad\textbf{(D)}\ 4\qquad\textbf{(E)}\ 5$

Solution

Sally made $0.55*20=11$ shots originally. Letting $x$ be the number of shots she made, we have $\frac{11+x}{25}=0.56$. Solving for $x$ gives us $x=\boxed{\textbf{(C)}\ 3}$

See Also

2004 AMC 8 (ProblemsAnswer KeyResources)
Preceded by
Problem 5
Followed by
Problem 7
1 2 3 4 5 6 7 8 9 10 11 12 13 14 15 16 17 18 19 20 21 22 23 24 25
All AJHSME/AMC 8 Problems and Solutions